LSAT and Law School Admissions Forum

Get expert LSAT preparation and law school admissions advice from PowerScore Test Preparation.

 Administrator
PowerScore Staff
  • PowerScore Staff
  • Posts: 8917
  • Joined: Feb 02, 2011
|
#26548
Complete Question Explanation
(The complete setup for this game can be found here: lsat/viewtopic.php?t=11086)

The correct answer choice is (E)

Given that H is a variable that causes fewer problems when it is ranked highly, and this is a Cannot Be True question, there is reason to suspect that the further “back” H is pushed, the more problems it will create. Indeed, when H is ranked sixth, then J must be ranked eighth, leaving no room for O. Hence, H cannot be ranked sixth and answer choice (E) is correct.
 alliewad95
  • Posts: 4
  • Joined: Feb 19, 2020
|
#84294
I am so lost on this question.... Can someone please do a step-by-step on this....
 Adam Tyson
PowerScore Staff
  • PowerScore Staff
  • Posts: 5153
  • Joined: Apr 14, 2011
|
#84352
Sure thing, alliewad95!

The question is asking where H CANNOT go, which means the four wrong answers will also be places that H CAN go. So we start by asking ourselves what we know about H .

We know H cannot go first, because only F or G go first. That would be too easy, and "first" is not an answer choice.

What else do we know about it?

H is before J, with exactly one variable between them. And J is before O. We can connect those ideas to get this sequence:

[H _ J] :longline: O

This sequence, with at least three things after H (whatever is in the blank space, J, and O) means that H cannot go 8th, 7th, or 6th. Is any of those an answer choice? Yes, answer E is "sixth"! That's our winner! There is no need to test or even think about the other answer choices, because we know with absolute certainty that H cannot be sixth.

This could also have been determined in the original diagram by including "not laws" for all the places that H, J, O, K, and L cannot go, but that's an awful lot of not laws, and so might not be worth the time and effort involved in writing them all down. Instead, just use the sequences to identify these kinds of restrictions.
 startedfromKZnowwehere
  • Posts: 7
  • Joined: Jul 11, 2021
|
#89220
Hi there,

This question clued me in that something must be wrong with my set-up, because although I guessed correctly that the answer was "cannot be sixth," I don't see why it can be anything other than 2nd or 4th.

I had made templates for this game:

F H K J G/M L O/M/G O/G
F K M/G H L J O/M/G O/G
G H K J M/G L O/M/F O/F
G K M/F H L J O/M/F O/F

Am I missing other possibilities here?

Thanks!
Kate
 Adam Tyson
PowerScore Staff
  • PowerScore Staff
  • Posts: 5153
  • Joined: Apr 14, 2011
|
#89233
Unfortunately, yes, Kate, you are missing a series of possible outcomes where F, G, or M could be 2nd. For example:

FGHKJMLO

The overlapping H_J and K_ _ L blocks can start in the 2nd position, as you have done, but can also start in the 3rd position, forcing O to be 8th.

As much as I love doing templates, in this game they aren't a great choice because there are so many of them. Once you infer that the blocks have to overlap and can only do so in two ways, that can be enough to get through the questions.

Get the most out of your LSAT Prep Plus subscription.

Analyze and track your performance with our Testing and Analytics Package.